Đến nội dung

IHateMath nội dung

Có 282 mục bởi IHateMath (Tìm giới hạn từ 26-05-2020)



Sắp theo                Sắp xếp  

#613994 $\sum_{a,b,c}\frac{a^2}{(b-c)^2+(b+c)...

Đã gửi bởi IHateMath on 10-02-2016 - 17:53 trong Bất đẳng thức và cực trị

Cho $a, b, c$ là các số thực dương. Tìm GTLN của

$P=\frac{a^2}{(b-c)^2+(b+c)a}+\frac{b^2}{(c-a)^2+(c+a)b}+\frac{c^2}{(a-b)^2+(a+b)c}.$




#614004 Rumani 2013 TST

Đã gửi bởi IHateMath on 10-02-2016 - 18:22 trong Tổ hợp và rời rạc

Cho tập $X=\{1,2,3,...1000\}$. Tìm số tự nhiên $n$ lớn nhất sao cho với 5 tập con bất kì chứa 500 phần tử $A_1, A_2, A_3, A_4, A_5$ của $X$, luôn tồn tại $1\le i,j \le 5$ mà $|A_i \cap A_j|\ge n$.




#614019 Rumani 2013 TST

Đã gửi bởi IHateMath on 10-02-2016 - 19:56 trong Tổ hợp và rời rạc

Cảm ơn bạn nhiều, lời giải này mình đọc rồi :D. Có ai có lời giải nào khác dùng cách lập bảng và đếm bằng hai cách không?




#614026 $f(f(x))=f(x)+x$

Đã gửi bởi IHateMath on 10-02-2016 - 20:09 trong Phương trình hàm

Tìm tất cả $f:\mathbb{R} \rightarrow \mathbb{R}$ thỏa mãn:

$f(f(x))=f(x)+x$.




#614029 TOPIC tổng hợp các bài toán tổ hợp rời rạc xuất phát từ các kì thi MO,các tạp...

Đã gửi bởi IHateMath on 10-02-2016 - 20:39 trong Tổ hợp và rời rạc

Bài 16 (IMOSL 1995):

Trong một cuộc họp, có $12k$ người tham gia, mỗi người bắt tay với đúng $3k+6$ người khác. Biết rằng với  bất kì một cách chọn cặp 2 người ta có số người bắt tay với cả 2 là như nhau. Hỏi có bao nhiêu người trong cuộc họp đó?




#614040 Rumani 2013 TST

Đã gửi bởi IHateMath on 10-02-2016 - 21:21 trong Tổ hợp và rời rạc

 

attachicon.gifCapture.PNG

ta có thể xem bảng như hình vẽ trên với $1$ ở ô $(i,j)$(hàng $i$ cột $j$) nghĩa là $i\in A_j$

$d(x)$ là số tập hợp mà phần tử $x$ có mặt

ta dễ thấy vài nhận xét sau(dễ chứng minh)

$\bullet \sum_{i=1}^{1000}d(i)=\sum_{j=1}^{5}\left | A_j \right |=2500$

$\bullet \sum_{i=1}^{1000}d^2(i)=\sum_{j\neq k}\left | A_j\cap A_k \right |=\sum \left | A_j \right |+2\sum_{j<k}\left | A_j\cap A_k \right |$

$\bullet \sum_{i=1}^{1000}d^2(i)\geq 2^2.500+3^2.500=6500$

$\Rightarrow \sum_{j<k}\left | A_j\cap A_k \right |=\frac{\sum_{i=1}^{1000}d(i)^2-\sum \left | A_j \right |}{2}\geq \frac{6500-2500}{2}=2000$

ở đây ta có $n=\max\left \{ i,j\in \left \{ 1,..,5 \right \}|min\left | A_i\cap A_j \right | \right \}$ do đó ta có điều sau

$C_5^2.n\geq 2000\Rightarrow n\ge 200$

 

Hình như cm này còn thiếu phần cm n=200 thỏa mãn thì phải. bạn có thể chỉ ra một cách lập bảng thỏa mãn ycbt cho n=200 ko?




#614518 Chứng minh rằng tích của 4 số nguyên dương liên tiếp không thể là tí...

Đã gửi bởi IHateMath on 12-02-2016 - 17:41 trong Số học

Chứng minh rằng tích của 4 số nguyên dương liên tiếp không thể là tích của 2 số nguyên dương liên tiếp.




#615564 $f(f(x))=f(x)+x$

Đã gửi bởi IHateMath on 17-02-2016 - 18:54 trong Phương trình hàm

Đặt $x_n= f(f(..(f(x))..)) $

Ta dễ có $x_{n+2} = x_{n+1} +x_{n} $

Suy ra $x_n = c_1.(\frac{1+\sqrt{5}}{2})^n + c_2.(\frac{1-\sqrt{5}}{2})^n$

Mặt khác, ta có $x_0=0 ; x_1=f(x) $

Từ đó tìm được $f(x)$ bằng cách tìm mối quan hệ giữa $c_1, c_2 $ và thay vào

Lưu ý, ta sẽ được 2 hàm khi thay lần lượt $c_1, c_2 $

Chỗ này mình ko hiểu. Theo cách đặt thì ta có $x_0=x_2-x_1=f(f(x))-f(x)=x$ chứ! Khi đó sẽ có vô số $c_1, c_2$ thỏa mãn :D




#623541 $(n+1)^7-n^7-1$

Đã gửi bởi IHateMath on 29-03-2016 - 22:36 trong Số học

Tìm số tự nhiên n sao cho $(n+1)^7-n^7-1$ là số chính phương




#623545 $(u_{n})$ có vô số số hạng dương và âm

Đã gửi bởi IHateMath on 29-03-2016 - 22:53 trong Dãy số - Giới hạn

IMO 2005 bài số 2




#623548 $(u_{n})$ có vô số số hạng dương và âm

Đã gửi bởi IHateMath on 29-03-2016 - 23:25 trong Dãy số - Giới hạn

Lời giải (THTT 343) Ta có các nhận xét sau:

(1) với i<j thì $u_i\ne u_j$, vì nếu $u_i=u_j$ thì các số trong tập {$u_1,u_2,...u_n$} khi chia cho j chỉ cho không quá j-1 số dư khác nhau 

(2) nếu i<j$\le n$ thì $|u_i-u_j|\le n-1$ vì nếu m=$|u_i-u_j|$>n-1 thì các số $u_1,u_2,...u_m$ cho ta nhiều nhất là m-1 số dư trong phép chia cho m 

(3) với mỗi $n\ge 1$, {$u_1,u_2,..u_n$} là tập gồm n số nguyên liên tiếp. thật vậy, với mỗi $n\ge 1$, kí hiệu i(n) và j(n) theo thứ tự là chỉ số của số lớn nhất và nhỏ nhất của dãy $u_1,u_2,...u_n$. theo (1) n số này nhận n giá trị đôi một khác nhau từ $u_{i(n)}$ tới $u_{j(n)}$ hay là |$u_{j(n)}-u_{i(n)}|\ge n-1$. Theo (2) lại có |$u_{j(n)}-u_{i(n)}|\le n-1$ nên |$u_{j(n)}-u_{i(n)}|$= n-1 và ${u_1,...,u_n}$ là tập {$u_{i(n)},u_{i(n)+1},...u_{j(n)}$}.

Từ (1) suy ra mỗi số nguyên xuất hiện nhiều nhất một lần trong dãy $(u_n)$. bây giờ xét một số nguyên x tùy ý. Do dãy có vô số số hạng âm, nên tồn tại $u_i<x$, và do có vô số số hạng dương nên tồn tại $u_j$>x. với $n\ge max(i,j)$ dãy $u_1,...,u_n$ chứa x, theo (3)




#623550 $x_{n+2}=\frac{(n-1)x_{n+1}+x_n}...

Đã gửi bởi IHateMath on 29-03-2016 - 23:53 trong Dãy số - Giới hạn

Cho dãy ($x_n$) xác định như sau: $x_1$=a, $x_2$=b, $x_{n+2}=\frac{(n-1)x_{n+1}+x_n}{n}$. Chứng minh ($x_n$) hội tụ và tính lim$x_n$ theo a và b




#630356 Định lý Zsigmondy và ứng dụng

Đã gửi bởi IHateMath on 30-04-2016 - 15:56 trong Tài liệu, chuyên đề, phương pháp về Số học

Tạp chí Mathematical Excalibur của Hong Kong với bài viết về định lý Zsigmondy và ứng dụng trong giải các bài toán khó.

Link: https://www.math.ust.hk/excalibur/v16_n4.pdf




#630547 Định lý Zsigmondy và ứng dụng

Đã gửi bởi IHateMath on 01-05-2016 - 16:26 trong Tài liệu, chuyên đề, phương pháp về Số học

Bạn có thể cho mình hỏi làm thể nào kiếm ra mấy cái tài liệu này được không ?

Google ấy thôi  :D




#631752 ĐỀ THI OLYMPIC CHUYÊN KHOA HỌC TỰ NHIÊN 2016

Đã gửi bởi IHateMath on 07-05-2016 - 15:15 trong Thi HSG cấp Tỉnh, Thành phố. Olympic 30-4. Đề thi và kiểm tra đội tuyển các cấp.

Câu 4: Mình làm như thế này:

Ta thấy rằng $2016=2^5.3^2.7$, mà $3^2.7=1+2(1+2(1+2(1+2(1+2.1))))$, vậy nên nếu $a_k=1$ là một số hạng của dãy thì 

$a_{2k}=2a_k+1=3$

$a_{4k}=2a_{2k}+1=7$

...

$a_{2^5.k}=63$
$a_{2(2^5.k+1)-1}=63.2$

...

$a_{2^5.(2^5.k+1)-1}=63.2^5=2016$.

Nếu ta cm được có vô hạn số hạng $a_{k}=1$ của dãy, thì như vậy, tồn tại vô hạn số hạng $a_{2^5(2^5.k+1)-1}$ tương ứng có giá trị $2016$.

Thật vậy, ta để ý rằng có vô hạn số hạng $0$ trong dãy vì từ số hạng một $a_i=0$ ta có được $a_{2i+1}=2a_i=0$, dẫn tới tồn tại một dãy vô hạn các số $0$. Điều này xảy ra nếu ta chọn $i=1$. Chú ý rằng dãy trên gồm toàn các số hạng có chỉ số lẻ, nên từ đẳng thức $a_{2n}=a_{2n+1}+1$, ta suy ra tồn tại một dãy con vô hạn của dãy đầu gồm toàn các số $1$, suy $đpcm$.

 

P/s: Vì là từ phép suy luận ngược, dựa vào công thức truy hồi của dãy nên có lẽ công thức trên cho ta tất cả các số hạng $2016$ của dãy. (?) Khi đó câu thứ hai có thể giải quyết khá dễ dàng, chỉ số nhỏ nhất đó là $2^5(2.2^5+1)-1=2079$.




#631785 ĐỀ THI OLYMPIC CHUYÊN KHOA HỌC TỰ NHIÊN 2016

Đã gửi bởi IHateMath on 07-05-2016 - 18:48 trong Thi HSG cấp Tỉnh, Thành phố. Olympic 30-4. Đề thi và kiểm tra đội tuyển các cấp.

Câu 3:

Mình giải quyết như sau:

Đặt $X=(K)\cap BC,Y=(L)\cap BC$.

OLYM-16.png

a) Dễ thấy $IE=IX$ và $IF=IY$, do đó $IK\bot EX, IL\bot FY$. Mặt khác, ta có

$\angle INB=\angle IMC=90^0-\frac{\angle BAC}{2}$ (do $AMN$ cân tại $A$),

$\angle NIB=\angle AMN-\angle NBI=90^0-\frac{\angle BAC}{2}-\frac{\angle ABC}{2}=\frac{\angle ACB}{2}=\angle ICX$. Vậy $\Delta NIB$ và $\Delta MCI$ đồng dạng, suy ra $\frac{IB}{IC}=\frac{IN}{CM}=\frac{BN}{IM}$, suy ra $\frac {IB^2}{IC^2}=\frac{BN}{CM}$ (Do $IM=IN$). Ngoài ra ta cũng dễ cm được hai tam giác $IFN$ và $EIM$ cũng đồng dạng, suy ra $\angle FIN=\angle EIM$ và $\frac{IE^2}{IF^2}=\frac{EM}{FN}\Rightarrow \frac{IB}{IC}=\frac{IF}{IE}(*)$.

Đến đây, ta lại có $\angle FYB=\angle FIB=\angle FIN+\angle NIB=\angle IEM+\angle ICM=\angle IXY+\angle IXE=\angle EXY$

$\Rightarrow EX||FY\Rightarrow I,L,K$ thẳng hàng.

b) Câu này ta sẽ cm $BYFQ, CXEP$ là các hình thang cân, suy ra $BY=FQ,XC=PE$. Kết hợp với $BY=XC$ sẽ có ngay $đpcm$.

Thật vậy, dễ thấy $KL$ là trục $đx$ của hình thang $FYXE$ suy ra nó là hình thang cân, tới đây dễ suy ra $BYFQ, CXEP$ là các hình thang cân. 

Ta gọi $T$ là giao điểm của tiếp tuyến chung trong của $(K), (L)$ với $BC$. Thế thì $TY.TB=TX.TC$. Ta có (dễ cm được) các cặp tam giác sau đồng dạng: $TIY,IBF$ và $TIX,ICE$. từ đó ta có các kết quả sau:

$\frac{TI}{IB}=\frac{TY}{IF}, \frac{TI}{IC}=\frac{TX}{IE}$,
$\Rightarrow \frac{IB}{IC}=\frac{TY.IF}{TX.IE}$, kết hợp với $(*)$ suy ra $TY=TX\Rightarrow BY=CX\Rightarrow FQ=EP (đpcm)$.




#631786 ĐỀ THI OLYMPIC CHUYÊN KHOA HỌC TỰ NHIÊN 2016

Đã gửi bởi IHateMath on 07-05-2016 - 18:50 trong Thi HSG cấp Tỉnh, Thành phố. Olympic 30-4. Đề thi và kiểm tra đội tuyển các cấp.

Mình nghĩ  $a_{2k}=2a_k+1=3$ chứ nhỉ ? 

Chết, sai rồi =), để mình sửa, cám ơn bạn. Nói chung không ảnh hưởng gì nhiều.




#631927 ĐỀ THI OLYMPIC CHUYÊN KHOA HỌC TỰ NHIÊN 2016

Đã gửi bởi IHateMath on 08-05-2016 - 13:28 trong Thi HSG cấp Tỉnh, Thành phố. Olympic 30-4. Đề thi và kiểm tra đội tuyển các cấp.

Một cách tiếp cận khác, nhẹ nhàng hơn cho câu 6 c):

OLYMPIC162.PNG

Đặt $J=DI\cap EF,S=BN\cap CM$. Kẻ đường thẳng vuông góc với $IL$, cắt $EF$ tại $X$, $DP$ tại $Y$. Dễ thấy $D,S,I,J$ thẳng hàng. Theo định lý Pascal, $B,N,K,S$ thẳng hàng, tương tự với $C,M,L,S$. Hơn nữa, theo bổ đề hình thang, $S$ là trung điểm $JD$, theo định lý con bướm $LX=LY$. Vậy $XY//JD\Rightarrow XY\bot BC$, tức là $IL//BC\Rightarrow K,I,L$ thẳng hàng $(đpcm)$.




#632353 $8(a+b)c-16c^2-(a-b)^2-32c\ge 0$

Đã gửi bởi IHateMath on 10-05-2016 - 21:30 trong Bất đẳng thức - Cực trị

Cho $a,b,c$ là các số thực thỏa mãn $-1\le a,b\le 1,-1\le c<0$ và $|a-b|\le -4c$. Chứng minh rằng $8(a+b)c-16c^2-(a-b)^2-32c\ge 0$.



#632358 $8(a+b)c-16c^2-(a-b)^2-32c\ge 0$

Đã gửi bởi IHateMath on 10-05-2016 - 21:39 trong Bất đẳng thức - Cực trị

bđt sai 
thử $a=1; b=2; c=3$ thì nó ra âm rồi  :lol:

Xin lỗi bạn mình gửi nhầm, đề đã được sửa. Bạn làm thử đi nhé :)



#632775 Bài bất đẳng thức VMO 1995?

Đã gửi bởi IHateMath on 12-05-2016 - 21:20 trong Bất đẳng thức - Cực trị

Mấy bạn nào giỏi link chỉ mình xem bài toán này có xuất xứ từ đâu vậy: (không cần lời giải vì có nhiều trên diễn đàn này rồi :) )

Cho $a,b,c$ là các số thực dương thỏa mãn $ab+bc+ca+abc\le 4$. CMR $a+b+c\ge ab+bc+ca$.

Một số bài viết trên forum chú thích thêm rằng đây là bài trong kỳ thi học sinh giỏi quốc gia 1995 (có bài ghi là 1996) nhưng trên thực tế mình không tìm thấy bài nào như vậy cả. Đây là link đề gốc, của bên AOPS (đảm bảo chính xác!!!):

VMO 1995https://www.artofpro...tional_olympiad

VMO 1996: https://www.artofpro...tional_olympiad

P/s: Tìm link gốc cho vui :)




#632900 Bài bất đẳng thức VMO 1995?

Đã gửi bởi IHateMath on 13-05-2016 - 17:03 trong Bất đẳng thức - Cực trị

Bài này là TST chứ bạn  :mellow:

Cũng không phải luôn, thực ra trong lúc đang kiểm tra đề VMO 1995 và 96 mình tiện thể ghé qua bên TST, nhưng cũng ko phải  :closedeyes:.




#632906 Bài đa thức British MO 1997

Đã gửi bởi IHateMath on 13-05-2016 - 17:41 trong Đa thức

(British MO 1997 R2 P3) Tìm số đa thức bậc $5$ với hệ số phân biệt lấy từ tập hợp {$1,2,3,4,5,6,7,8$} mà chia hết cho đa thức $x^2-x+1$.




#633083 $(2p+1)^3\leq (2k-1)^3 +\sum_{j=k}^{p}(2j-...

Đã gửi bởi IHateMath on 14-05-2016 - 17:09 trong Bất đẳng thức - Cực trị

với $k$ là số nguyên dương sao cho: $k\geq 5$ và số nguyên tố $p$ sao cho: $p\geq k$  

cmr: $(2p+1)^3\leq (2k-1)^3 +\sum_{j=k}^{p}(2j-1)^3$

Đầu tiên ta sẽ cmr $(2n+1)^3\leq 2.(2n-1)^3(\forall n\geq 5)(1)$. Thật vậy, điều này tương đương với $\sqrt[3]{2}.(2n-1)-(2n+1)\geq 0\Leftrightarrow 2(\sqrt[3]{2}-1)n\geq \sqrt[3]{2}+1\Leftrightarrow n\geq \frac{\sqrt[3]{2}+1}{2(\sqrt[3]{2}-1)}\Leftrightarrow n\geq 4,347...$, đúng.

Hiển nhiên với $p=k=5$ ta có $đpcm$ vì khi đó bất đẳng thức cần cm trở thành $(1)$.

Bây giờ, áp dụng bất đẳng thức trên cho $p\geq 7,k\geq 5$ với chú ý rằng $(2q-1)^3\leq 2.[2(q-1)-1]^3(\forall q\geq 6)$ (nếu ta thay $q$ bởi $q+1$ sẽ thu được bất đẳng thức $(1)$) ta có:

$(2p+1)^3\leq 2.(2p-1)^3\leq 2.[2(p-1)-1]^3+(2p-1)^3\leq 2.[2(p-2)-1]^3+[2(p-1)-1]^3+(2p-1)^3\leq...\leq (2(p-(p-k-1))-1)^3+\sum\limits_{j=p-(p-k-1)}^{p}{(2j-1)^3}\leq (2(p-(p-k))-1)^3+\sum\limits_{j=p-(p-k)}^{p}{(2j-1)^3}=(2k-1)^3+\sum\limits_{j=k}^{p}{(2j-1)^3}(đpcm)$.

P/s: Mình nghĩ bất đẳng thức này không xảy ra dấu bằng do $(1)$ không xảy ra dấu bằng. Hơn nữa không cần giả thiết là $p$ nguyên tố mà chỉ cần $p\geq k\geq 5$ là được. Liệu có thể làm chặt hơn không?




#633770 $4(\sum_{sym}a)^3 \geq 27(\sum_{cyc}...

Đã gửi bởi IHateMath on 17-05-2016 - 21:49 trong Bất đẳng thức và cực trị

Cho $a,b,c$ là các số thực dương. Chứng minh rằng $4(a+b+c)^3 \geq 27(a^{2}b+b^{2}c+c^{2}a+abc)$